Geometric Brownian motion












2












$begingroup$


This question is related to conditional expectation of a geometric Brownian motion.




The price of a stock is $10$ times a Geometric Brownian Motion with drift $mu = 0.05$ and $sigma = 0.2$.
Assume the stock price is $30$ at time $16$. What is the expected value of the stock price at time $25$?




The answer is $56.3283$



What formula should I use to get this answer?



According to the question, the stock price is $S(t) = 10{e^{x(t)}}$



Should I use $E[Z(t)] = {e^{mu t + {{sigma {t^2}} over 2}}}$?



Thanks in advance.










share|cite|improve this question











$endgroup$












  • $begingroup$
    If you plug the numbers into your suggested equation for the expected value, does it match?
    $endgroup$
    – Trurl
    May 23 '13 at 14:47










  • $begingroup$
    Can you cite the question? I'm looking for a text that discusses such topics. (I realize this post is three years old and I'm talking to a ghost.)
    $endgroup$
    – Andrew
    Feb 2 '16 at 20:33
















2












$begingroup$


This question is related to conditional expectation of a geometric Brownian motion.




The price of a stock is $10$ times a Geometric Brownian Motion with drift $mu = 0.05$ and $sigma = 0.2$.
Assume the stock price is $30$ at time $16$. What is the expected value of the stock price at time $25$?




The answer is $56.3283$



What formula should I use to get this answer?



According to the question, the stock price is $S(t) = 10{e^{x(t)}}$



Should I use $E[Z(t)] = {e^{mu t + {{sigma {t^2}} over 2}}}$?



Thanks in advance.










share|cite|improve this question











$endgroup$












  • $begingroup$
    If you plug the numbers into your suggested equation for the expected value, does it match?
    $endgroup$
    – Trurl
    May 23 '13 at 14:47










  • $begingroup$
    Can you cite the question? I'm looking for a text that discusses such topics. (I realize this post is three years old and I'm talking to a ghost.)
    $endgroup$
    – Andrew
    Feb 2 '16 at 20:33














2












2








2





$begingroup$


This question is related to conditional expectation of a geometric Brownian motion.




The price of a stock is $10$ times a Geometric Brownian Motion with drift $mu = 0.05$ and $sigma = 0.2$.
Assume the stock price is $30$ at time $16$. What is the expected value of the stock price at time $25$?




The answer is $56.3283$



What formula should I use to get this answer?



According to the question, the stock price is $S(t) = 10{e^{x(t)}}$



Should I use $E[Z(t)] = {e^{mu t + {{sigma {t^2}} over 2}}}$?



Thanks in advance.










share|cite|improve this question











$endgroup$




This question is related to conditional expectation of a geometric Brownian motion.




The price of a stock is $10$ times a Geometric Brownian Motion with drift $mu = 0.05$ and $sigma = 0.2$.
Assume the stock price is $30$ at time $16$. What is the expected value of the stock price at time $25$?




The answer is $56.3283$



What formula should I use to get this answer?



According to the question, the stock price is $S(t) = 10{e^{x(t)}}$



Should I use $E[Z(t)] = {e^{mu t + {{sigma {t^2}} over 2}}}$?



Thanks in advance.







probability






share|cite|improve this question















share|cite|improve this question













share|cite|improve this question




share|cite|improve this question








edited May 23 '13 at 13:53









Angela Richardson

5,25911733




5,25911733










asked May 23 '13 at 12:48









rexrex

212




212












  • $begingroup$
    If you plug the numbers into your suggested equation for the expected value, does it match?
    $endgroup$
    – Trurl
    May 23 '13 at 14:47










  • $begingroup$
    Can you cite the question? I'm looking for a text that discusses such topics. (I realize this post is three years old and I'm talking to a ghost.)
    $endgroup$
    – Andrew
    Feb 2 '16 at 20:33


















  • $begingroup$
    If you plug the numbers into your suggested equation for the expected value, does it match?
    $endgroup$
    – Trurl
    May 23 '13 at 14:47










  • $begingroup$
    Can you cite the question? I'm looking for a text that discusses such topics. (I realize this post is three years old and I'm talking to a ghost.)
    $endgroup$
    – Andrew
    Feb 2 '16 at 20:33
















$begingroup$
If you plug the numbers into your suggested equation for the expected value, does it match?
$endgroup$
– Trurl
May 23 '13 at 14:47




$begingroup$
If you plug the numbers into your suggested equation for the expected value, does it match?
$endgroup$
– Trurl
May 23 '13 at 14:47












$begingroup$
Can you cite the question? I'm looking for a text that discusses such topics. (I realize this post is three years old and I'm talking to a ghost.)
$endgroup$
– Andrew
Feb 2 '16 at 20:33




$begingroup$
Can you cite the question? I'm looking for a text that discusses such topics. (I realize this post is three years old and I'm talking to a ghost.)
$endgroup$
– Andrew
Feb 2 '16 at 20:33










1 Answer
1






active

oldest

votes


















0












$begingroup$

Under a geometric Brownian motion $$ dS_t = mu S_t {dt} + sigma S_t {dW_t} $$
where $mu = 0.05$ and $sigma = 0.2$,
the stock price is written $$ P_t = cS_t $$ where $c=10$.
Since $ mathbb{E}[S_t] = S_0 exp( mu t ) $, the expected stock value is
$$ mathbb{E}[P_t] = P_0 exp( mu t ) = 30 exp(0.05 t) $$
where we have shifted $t$ to "start" at $16$.
So at time $25$, we have $t=9$, so:
$$ mathbb{E}[P_t] = 30 exp(0.05 times 9) = 47.05 $$
which is wrong, apparently.



Either the question or I have made a mistake? Perhaps you can figure out which one?






share|cite|improve this answer









$endgroup$













    Your Answer





    StackExchange.ifUsing("editor", function () {
    return StackExchange.using("mathjaxEditing", function () {
    StackExchange.MarkdownEditor.creationCallbacks.add(function (editor, postfix) {
    StackExchange.mathjaxEditing.prepareWmdForMathJax(editor, postfix, [["$", "$"], ["\\(","\\)"]]);
    });
    });
    }, "mathjax-editing");

    StackExchange.ready(function() {
    var channelOptions = {
    tags: "".split(" "),
    id: "69"
    };
    initTagRenderer("".split(" "), "".split(" "), channelOptions);

    StackExchange.using("externalEditor", function() {
    // Have to fire editor after snippets, if snippets enabled
    if (StackExchange.settings.snippets.snippetsEnabled) {
    StackExchange.using("snippets", function() {
    createEditor();
    });
    }
    else {
    createEditor();
    }
    });

    function createEditor() {
    StackExchange.prepareEditor({
    heartbeatType: 'answer',
    autoActivateHeartbeat: false,
    convertImagesToLinks: true,
    noModals: true,
    showLowRepImageUploadWarning: true,
    reputationToPostImages: 10,
    bindNavPrevention: true,
    postfix: "",
    imageUploader: {
    brandingHtml: "Powered by u003ca class="icon-imgur-white" href="https://imgur.com/"u003eu003c/au003e",
    contentPolicyHtml: "User contributions licensed under u003ca href="https://creativecommons.org/licenses/by-sa/3.0/"u003ecc by-sa 3.0 with attribution requiredu003c/au003e u003ca href="https://stackoverflow.com/legal/content-policy"u003e(content policy)u003c/au003e",
    allowUrls: true
    },
    noCode: true, onDemand: true,
    discardSelector: ".discard-answer"
    ,immediatelyShowMarkdownHelp:true
    });


    }
    });














    draft saved

    draft discarded


















    StackExchange.ready(
    function () {
    StackExchange.openid.initPostLogin('.new-post-login', 'https%3a%2f%2fmath.stackexchange.com%2fquestions%2f400182%2fgeometric-brownian-motion%23new-answer', 'question_page');
    }
    );

    Post as a guest















    Required, but never shown

























    1 Answer
    1






    active

    oldest

    votes








    1 Answer
    1






    active

    oldest

    votes









    active

    oldest

    votes






    active

    oldest

    votes









    0












    $begingroup$

    Under a geometric Brownian motion $$ dS_t = mu S_t {dt} + sigma S_t {dW_t} $$
    where $mu = 0.05$ and $sigma = 0.2$,
    the stock price is written $$ P_t = cS_t $$ where $c=10$.
    Since $ mathbb{E}[S_t] = S_0 exp( mu t ) $, the expected stock value is
    $$ mathbb{E}[P_t] = P_0 exp( mu t ) = 30 exp(0.05 t) $$
    where we have shifted $t$ to "start" at $16$.
    So at time $25$, we have $t=9$, so:
    $$ mathbb{E}[P_t] = 30 exp(0.05 times 9) = 47.05 $$
    which is wrong, apparently.



    Either the question or I have made a mistake? Perhaps you can figure out which one?






    share|cite|improve this answer









    $endgroup$


















      0












      $begingroup$

      Under a geometric Brownian motion $$ dS_t = mu S_t {dt} + sigma S_t {dW_t} $$
      where $mu = 0.05$ and $sigma = 0.2$,
      the stock price is written $$ P_t = cS_t $$ where $c=10$.
      Since $ mathbb{E}[S_t] = S_0 exp( mu t ) $, the expected stock value is
      $$ mathbb{E}[P_t] = P_0 exp( mu t ) = 30 exp(0.05 t) $$
      where we have shifted $t$ to "start" at $16$.
      So at time $25$, we have $t=9$, so:
      $$ mathbb{E}[P_t] = 30 exp(0.05 times 9) = 47.05 $$
      which is wrong, apparently.



      Either the question or I have made a mistake? Perhaps you can figure out which one?






      share|cite|improve this answer









      $endgroup$
















        0












        0








        0





        $begingroup$

        Under a geometric Brownian motion $$ dS_t = mu S_t {dt} + sigma S_t {dW_t} $$
        where $mu = 0.05$ and $sigma = 0.2$,
        the stock price is written $$ P_t = cS_t $$ where $c=10$.
        Since $ mathbb{E}[S_t] = S_0 exp( mu t ) $, the expected stock value is
        $$ mathbb{E}[P_t] = P_0 exp( mu t ) = 30 exp(0.05 t) $$
        where we have shifted $t$ to "start" at $16$.
        So at time $25$, we have $t=9$, so:
        $$ mathbb{E}[P_t] = 30 exp(0.05 times 9) = 47.05 $$
        which is wrong, apparently.



        Either the question or I have made a mistake? Perhaps you can figure out which one?






        share|cite|improve this answer









        $endgroup$



        Under a geometric Brownian motion $$ dS_t = mu S_t {dt} + sigma S_t {dW_t} $$
        where $mu = 0.05$ and $sigma = 0.2$,
        the stock price is written $$ P_t = cS_t $$ where $c=10$.
        Since $ mathbb{E}[S_t] = S_0 exp( mu t ) $, the expected stock value is
        $$ mathbb{E}[P_t] = P_0 exp( mu t ) = 30 exp(0.05 t) $$
        where we have shifted $t$ to "start" at $16$.
        So at time $25$, we have $t=9$, so:
        $$ mathbb{E}[P_t] = 30 exp(0.05 times 9) = 47.05 $$
        which is wrong, apparently.



        Either the question or I have made a mistake? Perhaps you can figure out which one?







        share|cite|improve this answer












        share|cite|improve this answer



        share|cite|improve this answer










        answered Jan 13 at 20:10









        user3658307user3658307

        4,5933946




        4,5933946






























            draft saved

            draft discarded




















































            Thanks for contributing an answer to Mathematics Stack Exchange!


            • Please be sure to answer the question. Provide details and share your research!

            But avoid



            • Asking for help, clarification, or responding to other answers.

            • Making statements based on opinion; back them up with references or personal experience.


            Use MathJax to format equations. MathJax reference.


            To learn more, see our tips on writing great answers.




            draft saved


            draft discarded














            StackExchange.ready(
            function () {
            StackExchange.openid.initPostLogin('.new-post-login', 'https%3a%2f%2fmath.stackexchange.com%2fquestions%2f400182%2fgeometric-brownian-motion%23new-answer', 'question_page');
            }
            );

            Post as a guest















            Required, but never shown





















































            Required, but never shown














            Required, but never shown












            Required, but never shown







            Required, but never shown

































            Required, but never shown














            Required, but never shown












            Required, but never shown







            Required, but never shown







            Popular posts from this blog

            Mario Kart Wii

            What does “Dominus providebit” mean?

            Antonio Litta Visconti Arese